Euler's theorem

Leo79

New member
Joined
Jan 26, 2021
Messages
22
(Questions attached) my answer for 3a is:
Since p is a prime number, the only possible values of are 1 and p. The only way to have is if n is a multiple of p. There is only 1 multiple less than p. Therefore, the other p-1 numbers are all relatively prime to p. Hence, if and p is prime: φ(p)=p−1

I'm familiar with Fermat's theorem, although I'm a bit confused with how it translates in 3b - could I please have help with this? Is what I've put below on the right track?

IMG_2081.JPG
 

Attachments

  • Screen Shot 2021-02-09 at 8.09.43 pm.png
    Screen Shot 2021-02-09 at 8.09.43 pm.png
    69.6 KB · Views: 4
Top